konvergiert die reihe?

Neue Frage »

justdoit Auf diesen Beitrag antworten »
konvergiert die reihe?
hallo zuerst mal an alle,

ich hab hier eine Reihe vor mir, bei der ich überprüfen soll, ob sie konvergiert:

SUMME_k=1_bis.unendlich (e^n+n)/(3^n+ln(n))

was ich bereits versucht habe, ist das Quotientenkriterium, aber das scheint nicht zu funktionieren, da ich jeweils ne summe im nenner und zähler hab, da kürzt sich leider also ohne weiteres nichts weg.

weiterhin fallen auch leibnizkriterium (alterniert nicht) und das wurzelkriterium wohl raus( beim wurzelkriterium bin ich mir nicht ganz sicher,aber nur e und 3 haben den exponent n)
Dual Space Auf diesen Beitrag antworten »
RE: konvergiert die reihe?
Zitat:
Original von justdoit
SUMME_k=1_bis.unendlich (e^n+n)/(3^n+ln(n))

Wahrscheinlich ist k=n? Also geht es um



??
justdoit Auf diesen Beitrag antworten »

upps, ja. genau sieht die reihe aus Freude
WebFritzi Auf diesen Beitrag antworten »

Benutze das Majoranten-Kriterium. Schätze durch ein Vielfaches einer geom. Reihe ab.
justdoit Auf diesen Beitrag antworten »

wie soll ich dass denn in die form von q^n bringen?sodass ich überprüfen kann, ob der betrag von q<1 ist?
WebFritzi Auf diesen Beitrag antworten »

Wähle q = e/3.
 
 
justdoit Auf diesen Beitrag antworten »
RE: konvergiert die reihe?


oder? aber bringt mir das was? ich brauch doch entweder eine divergente minorante oder konvergente majorante?
tmo Auf diesen Beitrag antworten »

du hast sinnlos abgeschätzt.
wie wärs mit


den rest schaffst du alleine.
justdoit Auf diesen Beitrag antworten »

wenn man nach oben abschätzt, dann spekuliert man doch darauf, dass sie konvergiert oder?

woran macht ihr diese spekulation fest?
AD Auf diesen Beitrag antworten »

Zitat:
Original von justdoit
woran macht ihr diese spekulation fest?

An , oder anders geschrieben . Augenzwinkern
justdoit Auf diesen Beitrag antworten »

also muss ich nur nach den gliedern, die am schnellsten wachsen?

mich hat so bisschen gestört, dass e und 3 nur eine geringe differenz haben...so ca. 0.3, dafür wächste n schneller als ln(n), ich dachte, das würde diese minimaldifferenz kompensieren. anscheinend macht aber dieses 0.3 doch schon nen großen unterschied aus?
AD Auf diesen Beitrag antworten »

Eines sollte sich jedem, der Konvergenzbetrachtungen snstellt, zumindest im Laufe der Zeit einprägen:

Exponentialfunktionen wachsen schneller als Potenzfunktionen.

Mathematisch korrekt geschrieben:

Für alle positiven und beliebige reelle gilt

.

Das gilt auch für und . Augenzwinkern

Und Logarithmen wie wiederum wachsen langsamer als Potenzfunktionen mit positivem Exponent . Das ergibt sich aber aus der vorigen Aussage, wenn man substituiert.


Damit haben sich die beiden "Störsummanden" in Zähler und Nenner bei dir praktisch erledigt - ordentlich aufschreiben wie mit obiger Abschätzung sollte man es natürlich trotzdem.
tmo Auf diesen Beitrag antworten »

Zitat:
Original von justdoit
wenn man nach oben abschätzt, dann spekuliert man doch darauf, dass sie konvergiert oder?

woran macht ihr diese spekulation fest?


wenn man sich nicht sicher ist, kann man auch erstmal termumformung machen:



da die klammern gegen 1 konvergieren, ist wohl das entscheidende. und jetzt kann man sich sicher sein, dass eine abschätzung nach oben zum ziel führt, die man auch direkt schon an diese termumformung dranhängen kann:

.
statt 1.001 kann man da jede beliebige zahl > 1 hinschreiben. es zeigt immer die konvergenz.
AD Auf diesen Beitrag antworten »

Zitat:
Original von tmo
.

Um Irritationen seitens justdoit vorzubeugen, solltest du hier noch ein

Zitat:
für

dranhängen. Augenzwinkern
Neue Frage »
Antworten »



Verwandte Themen

Die Beliebtesten »
Die Größten »
Die Neuesten »